LSAT and Law School Admissions Forum

Get expert LSAT preparation and law school admissions advice from PowerScore Test Preparation.

 Administrator
PowerScore Staff
  • PowerScore Staff
  • Posts: 8917
  • Joined: Feb 02, 2011
|
#73940
Complete Question Explanation

Must Be True—CE. The correct answer choice is (C)

The causal relationship in this problem appears in the premise, and the argument is structured as
follows:

..... ..... Premise: Most antidepressant drugs cause weight gain.

..... ..... Premise: Dieting can help reduce the amount of weight gained while taking such antidepressants

..... ..... Conclusion: Some weight gain is unlikely to be preventable.

Note that the causal premise specifically states that “most” antidepressants cause weight gain,
not necessarily all antidepressants. Also, the second premise specifically refers to antidepressants
causing weight gain (the use of “such” indicates this). The second premise also indicates that the
amount gained can be reduced, not that dieting can stop weight gain. Perhaps the antidepressants
cause a twenty pound weight gain, but dieting can reduce that to a ten pound total gain.

The question stem is a Must Be True, and thus you must accept the stimulus information and find an
answer that is proven by that information.

Answer choice (A): This is an Exaggerated answer. The stimulus indicates that most antidepressants
cause weight gain, leaving open the possibility that some do not. This answer choice references any
antidepressant drug. Further, the stimulus does not address the role of a physician or the advisability
of prescribing certain drugs under certain conditions. The benefits of prescribing an antidepressant
that causes weight gain to an overweight patient may well outweigh the negatives (pun intended).

Answer choice (B): This is also an Exaggerated answer. The stimulus allows for antidepressants that
do not cause weight gain.

Answer choice (C): This is the correct answer. Some individuals taking antidepressants that cause
weight gain will gain weight even though dieting can reduce the amount of the gain.

Answer choice (D): This is an Opposite answer. The stimulus and correct answer both indicate that
people taking the weight gain-causing antidepressants will gain weight regardless of whether they
diet. Thus, the weight gain cannot be attributed to a lack of dieting.

Answer choice (E): This answer is too strong. Not all patients necessarily take antidepressants that
cause weight gain, so those that do not might not need to diet to maintain their weight. Also, some
patients who do take weight gain-causing antidepressants might be too thin for their own good and
could benefit from a weight gain-causing antidepressant.
 lsat2016
  • Posts: 59
  • Joined: May 29, 2016
|
#26616
Hello,

I had a small question regarding answer choice C.

The stimulus says that most antidepressant drugs cause weight gain and that some weight gain is unlikely to be preventable.

I chose C because it seems reasonable that at least some will gain weight. However, if this were a MBT question, would C also be a correct answer choice? For instance, isn’t it possible that if ALL people avoid taking the few antidepressant drugs that cause weight gain they wouldn’t gain weight? Or would it still be a correct answer choice because C the "likelihood" of occurrence always exists?

Thank you!
 Clay Cooper
PowerScore Staff
  • PowerScore Staff
  • Posts: 241
  • Joined: Jul 03, 2015
|
#26626
Hi Lsat2016,

Thanks for another good question.

I will deal with your questions individually, because I think we need to be careful about how we answer them.

If this were a MBT question, is C still correct?

Yes, because if it were not true that at least some patients taking antidepressant medications gain weight as a result of them, then the first premise (that most antidepressant drugs cause weight gain) would be false - which we know can't happen.

For instance, isn’t it possible that if ALL people avoid taking the few antidepressant drugs that cause weight gain they wouldn’t gain weight?

That is possible in reality, but it is not possible under what we are told in this stimulus, because (again) of the first premise. If everyone avoided taking the drugs that cause weight gain, then it wouldn't be true to say most drugs cause weight gain, and we know that can't be the case based on what we are told.

Or would it still be a correct answer choice because C the "likelihood" of occurrence always exists?

Likelihood is - in the hypothetical case that this stimulus were paired with a MBT question - irrelevant. Again, the first premise tells us for certain that some people take antidepressant drugs that cause weight gain. The fact that such drugs exist and, when used, are likely to cause weight gain would not be enough to make the first premise true; therefore, it must be the case that some people definitely do use these drugs and gain weight as a result.

Does that make sense?
 lsat2016
  • Posts: 59
  • Joined: May 29, 2016
|
#26635
Hello,

Yes. Thank you! That makes a lot of sense.
Just to clarify, I think I was confused because I interpreted the first sentence as "most antidepressants CAN cause weight gain". I was thinking that if there were 100 antidepressants and 80 drugs CAN cause weight gain but the majority of people use the other 20 drugs then this wouldn't necessarily be true in a MBT situation. However, you are saying that the statement "most antidepressants cause weight gain" already implies that it is true that some people have taken this drug and have gained weight as a result, right?

Also, I eliminated B because by saying that people who are trying to lose weight should not use antidepressants reflects an opinion that losing weight is a greater priority than getting rid of depression. However, it may be the case that they are suffering from a severe depression disorder and only a relatively small weight problem. Is this correct as well?

I think I'm focusing too much on the small details and am getting too confused as a result :hmm: Thank you so much for your explanations!
 David Boyle
PowerScore Staff
  • PowerScore Staff
  • Posts: 836
  • Joined: Jun 07, 2013
|
#26641
lsat2016 wrote:Hello,

Yes. Thank you! That makes a lot of sense.
Just to clarify, I think I was confused because I interpreted the first sentence as "most antidepressants CAN cause weight gain". I was thinking that if there were 100 antidepressants and 80 drugs CAN cause weight gain but the majority of people use the other 20 drugs then this wouldn't necessarily be true in a MBT situation. However, you are saying that the statement "most antidepressants cause weight gain" already implies that it is true that some people have taken this drug and have gained weight as a result, right?

Also, I eliminated B because by saying that people who are trying to lose weight should not use antidepressants reflects an opinion that losing weight is a greater priority than getting rid of depression. However, it may be the case that they are suffering from a severe depression disorder and only a relatively small weight problem. Is this correct as well?

I think I'm focusing too much on the small details and am getting too confused as a result :hmm: Thank you so much for your explanations!

Hello lsat2016,

Yes, the history of weight gain does imply that some drug users gained weight.
Answer B may in fact have a factor of people "weighing" their priorities, as you said. (Also, some drugs may not even cause a weight problem)

Hope this helps,
David
 DlarehAtsok
  • Posts: 50
  • Joined: Nov 18, 2015
|
#38187
Hi! Am I doing anything wrong, if I eliminate A, B and E on the basis that they are normative statements (given the "should) and the stimulus does not present a normative argument. Thanks in advance!
 Adam Tyson
PowerScore Staff
  • PowerScore Staff
  • Posts: 5153
  • Joined: Apr 14, 2011
|
#38573
Not only are you not doing anything wrong, DlarehAtsok, you are doing it exactly right! Normative statements - ones that express a value judgement rather than a factual one - have no place in the answer choices to a Must Be True question where the stimulus presented no such statements but instead was entirely factual. We talk about paying close attention to the language of the stimulus, including whether the argument is based on facts or opinions, certainties vs probabilities vs possibilities, the past or present or future, and other important distinctions. Well done for spotting the theme of these three wrong answers and tossing them into the trash where they belong!
 Hershel
  • Posts: 5
  • Joined: Aug 03, 2018
|
#49288
Clay Cooper wrote:Hi Lsat2016,

Thanks for another good question.

I will deal with your questions individually, because I think we need to be careful about how we answer them.

If this were a MBT question, is C still correct?

Yes, because if it were not true that at least some patients taking antidepressant medications gain weight as a result of them, then the first premise (that most antidepressant drugs cause weight gain) would be false - which we know can't happen.

For instance, isn’t it possible that if ALL people avoid taking the few antidepressant drugs that cause weight gain they wouldn’t gain weight?

That is possible in reality, but it is not possible under what we are told in this stimulus, because (again) of the first premise. If everyone avoided taking the drugs that cause weight gain, then it wouldn't be true to say most drugs cause weight gain, and we know that can't be the case based on what we are told.

Or would it still be a correct answer choice because C the "likelihood" of occurrence always exists?

Likelihood is - in the hypothetical case that this stimulus were paired with a MBT question - irrelevant. Again, the first premise tells us for certain that some people take antidepressant drugs that cause weight gain. The fact that such drugs exist and, when used, are likely to cause weight gain would not be enough to make the first premise true; therefore, it must be the case that some people definitely do use these drugs and gain weight as a result.

Does that make sense?
Thank you for your answer, but I would like to disagree (but I am not on any level to...).

The statement "Most ADs cause weight gain," does not necessarily imply that people are taking it. Perhaps the ADs were tested many years ago in a laboratory and proven to cause WG but no one any longer takes those ones, albeit the majority of ADs, which cause weight gain.

I would like to say (and I am not on any level to, yet) that "most strongly supports" is a slight variation of a Must Be True question. And, I found this advice on: https://www.manhattanprep.com/lsat/foru ... t3832.html.

Any concurrence?
 Who Ray
PowerScore Staff
  • PowerScore Staff
  • Posts: 27
  • Joined: Jul 31, 2018
|
#49481
Hi there Hershel!

I think you might be trying to argue yourself out of a correct answer. If this was a MBT question and you found yourself thinking about all these wider issues, that should be a clue that something is amiss. If it helps you to tackle questions by considering "most strongly supports" as distinct from MBT, then by all means do. But we have found that they do not warrant being treated as different groups for strategic purposes.

Have a good one!
Who Ray
User avatar
 broth99
  • Posts: 19
  • Joined: Mar 03, 2021
|
#91895
Hi Everyone,

I was just wondering why this problem was categorized as Must Be True. Because it seems in Chapter 4 Problem #4 (Every moral theory developed...), the answer key categorized the same question stem as Most Strongly Supported. I know a previous forum post had touched upon why the anti-depressant drug problem is MBT, but I would wondering if that could be expanded on and if someone could explain the difference between the "anti-depressant" question stem and the "moral theory" question stem.

Thanks,
Barath S.

Get the most out of your LSAT Prep Plus subscription.

Analyze and track your performance with our Testing and Analytics Package.